Class Description

Textbook: The Tools of Mathematical Reasoning

Real Numbers, Rational Numbers, and Integers

  • $\forall$ = “for all”, “for each”
  • $\exists$ = “for some”, “there exists”
  • $\mathbb{R}$ = Set of all Real Numbers

Properties of the Real Numbers

  1. $\forall x, y, z \in \mathbb{R} \ \ \ \ \ (x +y) + z = x + (y + z)$
  2. $\forall x \in \mathbb{R} x + y = y+ x$
  3. $\exists 0 \in \mathbb{R}\text{ such that }\forall x \in \mathbb{R}, x + 0 =x$
  4. $\forall x \in \mathbb{R} \exists (-x) \in \mathbb{R} \ \ \ \ \ \ \ \ \ x + (-x) = 0$
  5. $\forall x, y, z \in \mathbb{R} (x y ) z = x(yz)$ Associative
  6. $\forall x, y \in \mathbb{R}, xy = yx$ Commutative
  7. $\exists 1 \in \mathbb{R} $ such that $\forall x \in \mathbb{R}, x \cdot 1 = x $
  8. For any nonzero real number x, $\exists \frac{1}{x} \in \mathbb{R}$ such that $x \cdot \frac{1}{x} = 1$
  9. $\forall x, y, z \in \mathbb{R}, x (y+z) = xy + xz$
  10. $\forall x,y, z \in \mathbb{R} (y + z) x = yx + zx$
  11. The sum or product of any two real numbers is also a real number.

Proposition 1

Proposition

Let $x \in \mathbb{R}$, then $x \cdot 0 = 0$

Proof

By property 3, $0 + 0 = 0$.
Multiply both sides by x to get $x \cdot (0 + 0) = x \cdot 0$
$$ x \cdot 0 + x \cdot 0 = x \cdot 0$$

Property 4 $\exists -(x\cdot 0)$ such that $x \cdot 0 + - (x \cdot 0) = 0$
$$ (x \cdot 0 + x \cdot 0) + - (x \cdot 0) = (x \cdot 0) + -(x\cdot0)$$
$$ x \cdot 0 + (x \cdot 0 + (-x \cdot 0)) = 0$$
$$ x \cdot 0 + 0 = 0$$
$$ x \cdot 0 = 0$$

Proposition 2

Proposition

Let $a, b \in \mathbb{R}$. Assume that $ab = 0$. Then $a = 0$ or $b = 0$

Proof

Either $a = 0$ or $a \neq 0$

If $a = 0$, then there is nothing to prove. So we shall suppose $a \neq 0$, and try to prove $b =0$

$$ ab = 0 \text{ and } a \neq 0$$
By property 8 there is a real number $\frac{1}{a}$ so that $\frac{1}{a} = 1$
$$ \frac{1}{a} (ab) = \frac{1}{a} \cdot 0$$
$$ \left(\frac{1}{a} \cdot a\right) b = \frac{1}{a} \cdot 0 $$
$$ 1 \cdot b = 0 $$
Hence $b = 0$. So $a = 0$ or $b = 0$.

Set of Integers

$\mathbb{R}$ has a subset called the set of integers, denoted by $\mathbb{Z}$
$$ \mathbb{Z} = \{0,\pm 1, \pm 2, \pm 3, \pm 4, \pm 5… \}$$

  • $\mathbb{Z}$ is closed under addition and multiplication
  • If $x \in \mathbb{Z}$, and $y \in \mathbb{Z}$, then $x + y \in \mathbb{Z}$ and $xy \in \mathbb{Z}$

Abbreviations

In $\mathbb{R}$, we can abbreviate $a\cdot \frac{1}{b}$ by $\frac{a}{b}$ if $b\neq 0$. $a + (-b)$ by $a-b$.

Set of Rational Numbers

$$ \mathbb{Q} = \{ \frac{a}{b} : a, b, \in \mathbb{Z} \text{ and } b \neq 0\}$$

  • $\mathbb{Q}$ is a subset of $\mathbb{R}$

Question

  • Suppose $x, y \in \mathbb{Q}$. Is it true that $x + y \in \mathbb{Q}$?

Answer

Since $x \in \mathbb{Q}$, there are integers $a$ and $b$ ($b\neq 0$) so that $x = \frac{a}{b}$. Similarly there are integers $c$ and $d$ ($d \neq 0$) so that $y = \frac{c}{d}$.
$$ \frac{x}{y} = \frac{a}{b} + \frac{c}{d} = \frac{ad}{bd} + \frac{bc}{bd} $$
$$ = \frac{ad + bc}{bd}$$

  • $b\neq 0$ and $d \neq0$, so by Prop 2, $bd \neq 0$.

  • $b \in \mathbb{Z} \text{ and } d \in \mathbb{Z}$, so $bd \in \mathbb{Z}$ by closure.

  • $a \in \mathbb{Z}$ and $d \in \mathbb{Z}$, so $ad \in \mathbb{Z}$ by closure.

  • $b \in \mathbb{Z}$ and $c \in \mathbb{Z}$, so $bc \in \mathbb{Z}$ by closure.

  • One more use of closure shows that $ad + bc \in \mathbb{Z}$

  • Hence $x + y$ is the ratio of two integers, with non-zero denominator.

  • Thus $x + y \in \mathbb{Q}$. QED

  • Let $x, y \in \mathbb{Q}$. Is it true that $xy \in \mathbb{Q}$? Answer: Yes.

6.1: Division Algorithm and Well Ordering Principle

Let $\mathbb{N} = \{0, 1, 2, 3, 4, …\}$

Well Ordering Principle

Every nonempty subset of sequence of non-negative integers has a least element. Example: $\mathbb{N}$ has a least element.

Division Algorithm

Statement

Let $a \in \mathbb{Z}, b \in \mathbb{Z}$ and let $b > 0$

There is a unique pair of integers $q$ and $r$ such that $a = bq +r, \ \ \ \ 0 \le r < b$

q is the quotient and r is the remainder

Proof

Existence

First we need to prove that $q$ and $r$ exist such that $a = bq +r \ \ \ \ 0 \le r < b$. After that we’ll prove that there is only one pair $q$ and $r$ that is unique for a certain $a$ and $b$.

Define a set, $S$, as follows

$$ S = \{a-bk: k \in \mathbb{Z}, a - bk \in \mathbb{N}\}$$

In order to use the Well Ordering Principle (WOP), we need to show that $S \ne \emptyset$.

$$ \exists k : k < \frac{a}{b}, k \in \mathbb{Z}$$
There is at least one integer less than $\frac{a}{b}$ since $b > 0$. If $a < b$, then the least integer would be 0.

$$ k < \frac{a}{b} \implies a - bk > a - b\left(\frac{a}{b}\right)$$
$$ a-bk > a - a$$
$$ a-bk > 0$$

Since $k \in \mathbb{Z}$ and $a, b \in \mathbb{Z}$, by closure, $a - bk \in \mathbb{Z}$. Since $a - bk > 0$, $a - bk \in S$. Since $S \subseteq \mathbb{N}$, by the WOP, $S$ has a least element.

Now we need to prove that $0 \le r < b$.

Let $r$ denote the least element of $S$. Then since $r \in S, \exists q \in \mathbb{Z}: r = a - bq$

Since $r \in S$, we know that $r \in \mathbb{N} = \{0, 1, 2, 3, …\}$, so $r \ge 0$.

Now we just need to prove the second part of the inequality, $r < b$. Use a Proof by contradiction and assume the opposite: $r \ge b$.

$$ r \ge b \implies r - b \ge 0$$
$$ \implies (a-bq) - b \ge 0$$
$$ \implies r-b = a - b(q + 1) \ge 0$$

If $r-b \ge 0$, then $r - b\in S$. If that were true, $r-b$ would be the least element in S, since $b \in \mathbb{Z}, b > 0$ and $r \ge 0$. This is a contradiction since $r$ is the least element in $S$, and not $r-b$.

Thus $0 \le r < b$.

Uniqueness

Prove that $r$ and $q$ are unique solutions to $a = bq +r$.

Define $r'$ and $q'$
$$ r’ = bq’ + t, \ \ \ \ 0 \le r’ < b$$
Our goal is to show that $r’ = r$ and $q’ = q$.

$$ bq + r = a = bq’ + r'$$
$$ r - r’ = b(q’ - q)$$

This means that $b | (r - r’)$, so b is a multiple of $r-r'$.

Also since $0 \le r’ < b$ and $0 \le r < b$, $-b < r-r’ < b$.
$$ -b < r - r’ < b$$
$$ -b < b(q’ - q) < b$$
$$ -1 < q’ - q < 1$$
Since $q’ - q \in \mathbb{Z}$, $q’-q = 0$. So $q’ = q$.
$$ bq + r = bq’ + r'$$
$$ r = r'$$

Using the Division Algorithm

The Division Algorithm can reduce proofs about integers to finitely many cases.

We say that $a | b$, a divides b, if b is a multiple of a. $$ \exists q: b = aq + 0, \ \ q \in \mathbb{Z}$$

Example

Prove that the square of any integer can be expressed in one of the following forms: $4k +1$ or $4k + 0$

$$n \in \mathbb{Z}, n = (4q + r), r\in \{0, 1, 2, 3\}$$

Case r = 0

$$ n^2 = (4q+0)^2 = 16q^2$$
$$ n^2 = 4(4q^2)$$
$$n^2 = 4k + 0 ,\ \ \ \ \ \ k = 4q^2 \in \mathbb{Z} \text{ by closure}$$

Case r = 1

$$ n^2 = (4q+1)^2 = 16q^2 + 8q + 1$$
$$ n^2 = 4(4q^2 + 2q) + 1$$
$$n^2 = 4k + 1 ,\ \ \ \ \ \ k = 4q^2 + 2q \in \mathbb{Z} \text{ by closure}$$

Case r = 2

$$ n^2 = (4q+2)^2 = 16q^2 + 16q + 4$$
$$ n^2 = 4(4q^2 + 4q + 1) + 0$$
$$n^2 = 4k + 0 ,\ \ \ \ \ \ k = 4q^2 + 4q + 1\in \mathbb{Z} \text{ by closure}$$

Case r = 3

$$ n^2 = (4q+3)^2 = 16q^2 + 24q + 9$$
$$ n^2 = 16q^2 + 24q + 2(4) + 1$$
$$ n^2 = 4(4q^2 + 6q + 2) + 1 $$
$$n^2 = 4k + 1 ,\ \ \ \ \ \ k = 4q^2 + 6q + 2 \in \mathbb{Z} \text{ by closure}$$

Thus since $n^2 = 4k + 1$ or $n^2 = 4k +0$ for all values of r, we have proven that the square of any integer can be expressed in just those forms.

1.1: Language of Proofs

Proposition

  • Proposition: Sentence which has exactly one truth value
    • Not a proposition:
      • $4 \cdot 5$ (no verb in sentence)
      • $18 | n$ (The truth of falsity depends on the value of $n$)
  • Predicate: A sentence whose truth value depends on values assigned to free variables
  • Propositions can be combined with logical operators like AND, OR, NOT, IF, THEN, IFF

Implication

$$P \implies Q$$

  • P implies Q

  • If P then Q

  • P is sufficient for Q

  • P only if Q

  • Q when P

  • Q if P

  • P is called the hypothesis of the implication

  • Q is called the conclusion

$$
\begin{array}{|c c|c|}
% |c c|c| means that there are three columns in the table and
% a vertical bar ’|’ will be printed on the left and right borders,
% and between the second and the third columns.
% The letter ’c’ means the value will be centered within the column,
% letter ’l’, left-aligned, and ’r’, right-aligned.
p & q & p \implies q \\ % Use & to separate the columns
\hline % Put a horizontal line between the table header and the rest.
T & T & T \\
T & F & F \\
F & T & T \\
F & F & T \\
\end{array}
$$
$$ p\implies q \equiv \neg p \lor q $$

  • Converse: $Q \implies P$

  • Contrapositive: $\neg Q \implies \neg P$

    • Has the same truth table as the regular $P \implies Q$
  • Example

    • Theorem:
      $$ \text{If } \sum_{n=1}^\infty a_n \text{ converges, then } \lim_{n\to\infty} a_n = 0 $$

    • Contrapositive of Theorem:
      $$ \text{If } \lim_{n\to\infty} a_n \neq 0 \text{, then } \sum_{n=1}^\infty a_n \text{ diverges (Divergence Test)}$$

    • Converse:
      $$ \text{If } \lim_{n\to\infty} a_n = 0 \text{, then } \sum_{n=1}^\infty a_n \text{ converges (Not True)}$$

DeMorgan’s Laws & Formulas

Let $P$ and $Q$ be propositions

  1. $\neg (P \lor Q)$ is logically equivalent to $(\neg P) \land (\neg Q)$
  2. $\neg (P \land Q)$ is logically equivalent to $(\neg P) \lor (\neg Q)$

Proof of 1.
$$
\begin{array}{|c c|c|c|}
p & q & p \lor q & \neg(p \lor q) \\ % Use & to separate the columns
\hline % Put a horizontal line between the table header and the rest.
T & T & T & \mathbf{F} \\
T & F & T & \mathbf{F} \\
F & T & T & \mathbf{F} \\
F & F & F & \mathbf{T} \\
\end{array}
$$

$$
\begin{array}{|c c|c|c|c|}
p & q & \neg p & \neg q & (\neg p) \land (\neg q) \\ % Use & to separate the columns
\hline % Put a horizontal line between the table header and the rest.
T & T & F & F & \mathbf{F} \\
T & F & F & T & \mathbf{F} \\
F & T & T & F & \mathbf{F} \\
F & F & T & T & \mathbf{T} \\
\end{array}
$$

Quantifiers

  • Universal Quantifier: $\forall$ = “for any”, “for all”, or “for each”
  • Existential Quantifier: $\exists$ = “there exists”, “for some”
    • $\exists!$ = “there is a unique” or “there is exactly one”
  • Examples
    1. Some positive integers are both perfect squares and perfect cubes
    • $\exists x \in \mathbb{Z}^+ \vert (\exists y \in \mathbb{Z} \vert x = y^2) \land (\exists w \in \mathbb{Z} \vert x= w^3)$

Negating Quantifiers

$$ \neg(\forall x P(x)) \iff \exists x \neg P(x)$$
$$ \neg(\exists x P(x)) \iff \forall x \neg P(x)$$
$$ \neg(\exists x P(x)) \iff \nexists \ \ x \ \ P(x)$$

Example

$$ \forall x \in \mathbb{R} \ \exists k \in \mathbb{R}\ (x > 0 \land x=k^2)$$
The negation of the above would be
$$ \neg(\forall x \in \mathbb{R} \ \exists k \in \mathbb{R} \ (x > 0 \land x=k^2))$$
$$ \exists x \in \mathbb{R} \ \neg(\exists k \in \mathbb{R} \ (x > 0 \land x=k^2))$$
$$ \exists x \in \mathbb{R}\ \forall k \in \mathbb{R} \ \neg(x > 0 \land x=k^2))$$
Use Demorgan’s laws
$$ \exists x \in \mathbb{R}\ \forall k \in \mathbb{R}\ \neg(x > 0) \lor \neg(x=k^2)$$

Since $P \implies Q \equiv \neg P \lor Q$
$$ \exists x \in \mathbb{R}\ \forall k \in \mathbb{R}\ (x > 0 \implies x\neq k^2)$$

1.2: Direct Proofs

We can assume $P$ to be true and show that $Q$ is true.

2.1: More on Direct Proofs

Basic Properties of Real Numbers

For all real numbers $a$, $b$, and $c$

  1. Closure under addition and multiplication: $a + b \in \mathbb{R}, ab \in \mathbb{R}$
  2. Associative Properties: $(a+b) +c = a+(b+c)$ and $(ab)c = a(bc)$
  3. Commutative Properties: $a + b = b+a$ and $ab = ba$
  4. Distributive Properties: $a(b + c) = ab + ac$
  5. Identities: $0 \neq 1, a + 0 = a, a \cdot 1 = a, a \cdot 0 = 0$
  6. Additive Inverses: $\exists! -a = -1 \cdot a: a + (-a) = 0$
  7. Subtraction: $b-a$ is defined to equal $b + (-a)$
  8. Multiplicative Inverses: $a \neq 0 \implies \exists! a^{-1} = \frac{1}{a}: a\cdot a^{-1} = a \cdot \frac{1}{a} = 1$
  9. Division: $a \neq 0 \implies \frac{b}{a} = b \cdot \frac{1}{a}$

Additional Axioms for Real Numbers

  1. $\forall a, b \in \mathbb{R}$ exactly one of $a <b$, $a=b$, or $a > b$ is true
  2. $\forall a, b , c\in \mathbb{R} \ \ \ (A < b) \implies (a + c < b+c)$
  3. $\forall a, b, c \in \mathbb{R} \ \ (a < b \land c > 0) \implies ac < bc$
  4. $\forall a, b, c \in \mathbb{R} \ \ (a < b \land c < 0) \implies ac > bc$
  5. $\forall a, b, c \in \mathbb{R} \ \ (a < b \land b < c) \implies a < c$ (Transitive)

Counterexamples

The negation of $\forall x \in S, P(x)$ is $\exists x \in S, \neg P(x)$.
To disprove $\forall x \in S, P(x)$, we need only find one example of an x for which P(x) is false.

Prime Numbers

Definition: $n \in \mathbb{Z}^+$ is prime the following is true:
$$n\neq 1 \land (x \in \mathbb{Z}^+ \land x | n) \implies (x = 1 \lor x = n)$$

Double Implications

  • To prove $P \iff Q$, you need to prove both $P \implies Q$ and $Q \implies P$

Uniqueness

To prove a property is unique for a certain object, assume that there are two objects with this property $x$ and $y$ and show that they are the same.

Example

Prove the following statement:
$$ \exists x \in \mathbb{R} \forall y \in \mathbb{R} x + y = y+x = y$$
We will prove that this additive property is unique to just one number.

  1. Let $a\in\mathbb{R}$ be such that $a + w = w+a = w, \ \ \forall w \in \mathbb{R}$
  2. Let $b\in\mathbb{R}$ be such that $b + w = w+b = w, \ \ \forall w \in \mathbb{R}$
  • $a + b = b$ by line 1
  • $a + b = a$ by line 2
  • Thus we can conclude that $a = b$ and that only one unique number has the additive property we were interested in

2.2: Indirect Proofs

Note: $\therefore$ means therefore and $\because$ means because

Proof by Contradiction

To prove statement $P$ is to assume $\neg P$ and then derive a statement of the form $Q \land (\neg Q)$. If $\neg P$ implies a falsehood, then $\neg (\neg P)$, which means $P$ itself is true.

Proof by Contrapositive

$$ \neg Q \implies \neg P \iff P \implies Q $$

2.3: Two Important Theorems

Theorem: $\ \sqrt{2}\text{ }$ is irrational

Let $\sqrt{2}$ denote the positive real number $y$ satisfying $y^2 = 2$.

Suppose BWOC (By way of contradiction),
$$ \sqrt{2} \in \mathbb{Q} \implies \exists a, b \in \mathbb{N}: \sqrt{2} = \frac{a}{b}$$

If we cancel common factors so no integer divides both $a$ and $b$ except $\pm 1$, we can say $\sqrt{2} = \frac{p}{q}$, where $p, q \in \mathbb{Z}$ and they share no common factors.

$$ 2 = \frac{p^2}{q^2} \implies 2 q^2 = p^2 $$
$$ 2 | p^2 \implies 2 | p, \ p = 2t, \ t \in \mathbb{Z}$$
$p$ is even since $p^2$ is even.

$$ 2q^2 = (2t)^2$$
$$ 2q^2 = 4t^2$$
$$ q^2 = 2t^2 \implies 2 | q^2 \implies 2 | q$$

$2 | p \land 2 | q$, contradicting our choice of $p$ and $q$ sharing no common factors.

Thus $\sqrt{2} \notin \mathbb{Q}$

Greatest Common Divisor (GCD)

Let $a, b \in \mathbb{Z}$ and $a \neq 0 \land b \neq 0$.
$$S = \{ x \in \mathbb{Z} : x | a \land x | b \}$$
$S$ is the set of common integer divisors of $a$ and $b$

$$ \forall a, b \in \mathbb{Z}: 1 \in S \implies S \neq \emptyset$$
If $|k| > max(a, b)$, then $k \nmid a \land k \nmid b$, so $k \notin S$
So $S$ is finite and $S$ has a maximum element.

Define GCD as the following:
$$ gcd(a, b) = max(S)$$

If $a = b = 0$, we define $gcd(0, 0) = 0$

Bézout’s Lemma

Lemma: Let $a, b \in \mathbb{Z}$. Then $\exists m, n \in \mathbb{Z}: gcd(a, b) = am + bn$

Proof

Case 1
$$a = b = 0$$
$$gcd(0, 0) = 0 = a \cdot 0 + b \cdot 0$$
Case 2
$$ a = 0 \land b \neq 0$$
$$ gcd(0, b) = |b|$$
Case 3
$$ a \neq 0 \land b = 0$$

  • Same as Case 2

Case 4
$$ T = \{x \in \mathbb{N}: \exists u, v \in \mathbb{Z} : x = au + bv\}$$
$$ a^2 + b^2 = a(a) + b(b) > 0$$
$$ a^2 + b^2 \in \mathbb{T}$$
$$ T \neq \emptyset \land T \in \mathbb{N} \implies d = min(T)$$

  • By the Well Ordering Principle (WOP), the set $T$ has a minimum element.

  • Now we show that $d | a$
    $$ a = dq + r \ \ \ q, r \in \mathbb{Z}, \ \ \ 0 \le r < d$$
    $$ d \in T \implies \exists s, t \in \mathbb{Z}: d = as + bt$$

  • If $r \neq 0$, then $r \in \mathbb{N}$ and $r = a - dq = a - (as + bt)q = a(1 - qs) + b(-tq)$, so that $r \in T$

  • But $r < d$, so this would contradict that $d = min(T)$, since $r < d$. Hence $r = 0$ must be true and $d | a$. Similarly $d | b$

  • We’ve shown that $d|a \land d | b$. Now we show that $d$ is the greatest of the common divisors of $a$ and $b$. Let $c$ be another common divisor of $b$ and $a$
    $$ c \in \mathbb{Z}: c | a \land c| b$$
    $$ \exists w, z \in \mathbb{Z}: q = cz, \ b = cw$$
    $$ d = as + bt = (cz) s + (cw) t$$

  • Since $d > 0$, this means $c \le d$ since $c$ is a divisor of $d$. So $d$ is the greatest common divisor of $a$ and $b$

Euclid’s Lemma

Let $a, b \in \mathbb{Z}$ and let $p $ be a prime number. If $p | ab$, then $p | a\lor p | b$

Proof

If $p | a$, then we are done, so suppose $p \nmid a$ and try to show $p | b$.
If $p \neq a$ and $p$ is prime, then $gcd(a, p) = 1$
By Bézout’s Theorem, $\exists m, n \in \mathbb{Z}$ so that $am + pn = 1$
$$ \exists x \in \mathbb{Z}: px = ab$$
$$ b(am + pn) = 1b$$
$$ (ba)m + pnb = b$$
$$ (px)m + pnb = b $$
$$ p(xm + nb) = b $$
$xm + nb \in \mathbb{Z}$, so $p | b$, as desired.

2.4: Statements Including Mixed Quantifiers

$\forall \exists$

To prove a statement of the form $\forall x \in S \ \ \exists y \in T \ \ P(x, y)$, we let $x$ be an arbitrary element of S, and we try to find $y \in T$ for which $P(x, y)$ is true.

Example

Let $\mathbb{R}^{+}$ = set of positive real numbers.
Prove that $\forall \epsilon \in \mathbb{R}^{+} \ \ \exists \delta \in \mathbb{R}^+$ such that $|4x - 12| < \epsilon$, whenever $0 < |x -3 | < \delta$.

Proof

Let $\epsilon \in \mathbb{R}^+$ be given.
Choose $\delta = \frac{\epsilon}{4}$. Now, if $0 < |x - 3| < \frac{\epsilon}{4}$, then $|4x - 12| = 4 | x - 3| < 4\cdot \frac{\epsilon}{4} = \epsilon$
So that $|4x - 12 < \epsilon$ as desired.

$\exists \forall$

To prove a statement of the form $\exists x \in S \ \ \forall y \in T \ \ P(x, y)$, we carefully choose $x \in S$ and try to show that $\forall y \in T$, $P(x, y)$ holds.

Example

Prove that $\exists x \in \mathbb{Z}$ such that $\forall y \in \mathbb{R}, \ xy \in \mathbb{Z}$.

Proof

Put $x = 0\in \mathbb{Z}$, $\forall y \in \mathbb{R}, x\cdot y = 0 \cdot y = 0 \in \mathbb{Z}$, as desired.

Changing the order of mixed quantifiers changes the meaning of the statement.

Epsilon Delta Definition of Limit

We say $\lim_\limits{x\to c} f(x) = L$ if $\forall \epsilon \in \mathbb{R}^+ \exists \delta \in \mathbb{R}^+$ such that $(0 < |x - c| < \delta) \implies (|f(x) - L|) < \epsilon$

Triangle Inequality

$$ \forall x, y \in \mathbb{R}, | x + y | \le |x| + |y|$$

Proposition

Suppose $\lim\limits_{x\to c} f(x) = L$ and $\lim\limits_{x \to c} g(x) = M = L$, then $\lim\limits_{x \to c} \left( f(x) + g(x) \right) = \lim\limits_{x\to c} f(x) + \lim\limits_{x \to c} g(x)$

Proof

We want $\forall \epsilon \in \mathbb{R}^+ \exists \delta \in \mathbb{R}^+$, so that $(f + g)(x) - (L + M) < \epsilon$ whenever $ 0 < | x -c < \delta$. We let $\epsilon \in \mathbb{R}^+$ be given.

Since $\lim\limits_{x \to c} f(x) = L$, $\exists \delta \in \mathbb{R}$, so that $f(x) -L < \frac{\epsilon}{2}$ whenever $0 < |x - c| < \delta_1$
Since $\lim\limits_{x \to c} f(x) = M$, $\exists \delta \in \mathbb{R}$, so that $f(x) -M < \frac{\epsilon}{2}$ whenever $0 < |x - c| < \delta_2$

Let $\delta = min(\delta_1, \delta _2$
$$ \mid (f(x) + g(x)) - (L +M) \mid = | f(x) + g(x) - L - M| $$
$$| (f(x) - L) + (g(x) - M)| \le | f(x) - L| + | g(x) - M| < \frac{\epsilon}{2} + \frac{\epsilon}{2} = \epsilon $$

3.1: Mathematical Induction

Weak Induction Law

Suppose $S \subseteq \mathbb{N}$ has the following 2 properties

  1. $1 \in S$
  2. $n \in S \implies n + 1 \in S$

Then $S = \mathbb{N}$

Proof

Let $T$ = set of positive integers NOT in $S$
Suppose BWOC, that $T \neq \emptyset$
$T \subseteq \mathbb{N}$, so by the Well-Ordering Principle, $T$ has a least element, $m$.
By 1. $m \neq 1$, so $m > 1$. Then $m - 1 \in \mathbb{N}$
$m -1 < m$, and $m$ is the least element of $T$, so $m -1 \in S$
By 2. $(m - 1) + 1 \in S$, i.e. $m \in S$, but this contradicts our choice of $m$.
So we must have that $T$ is empty, and $S = \mathbb{N}$

Example 1

  • Prove that the following is true:
    $$ \sum_{i=1}^{n} \frac{n(n+1)}{2} \ \ \forall n \in \mathbb{N}$$

  • Base Case: $n = 1$
    $$ \sum_{n=1}^{1} i = 1 = \frac{1 \cdot (1 + 1)}{2} = \frac{2}{2} = 1$$

    so the result is true when n = 1

  • Inductive Hypothesis: Suppose that $ \sum_{i=1}^{m} i = \frac{m(m+1)}{2}$ for some $m \in \mathbb{N}$

    • We will prove that $\sum_{i=1}^{m+1} i = \frac{(m+1)((m+1) + 1)}{2} = \frac{(m+1)(m+2)}{2}$

    $$ \sum_{i=1}^{m+1} = \left( \sum_{i=1}^{m} i \right) + (m+1) = \frac{m(m+1)}{2} + (m+1) = \frac{(m+1)(m+2)}{2}$$

    • So by induction $ \sum_{i=1}^{n} i = \frac{n(n+1)}{2} \ \ \forall n \in \mathbb{N} $

Example 2

  • Prove that for all integers $n$ with $n \ge 4$, we have $2^n < n!$

  • We proceed by induction

  • Base Case: $n = 4$
    $$ 2^4 = 16$$
    $$ 4! = 1 \cdot 2 \cdot 3 \cdot 4 = 24$$

  • Inductive Hypothesis: Suppose that $2^m < m!$ for some integer $m \ge 4$.

  • We show that $2^{m+1} < (m+1)!$
    $$ 2^{m+1} = 2^m \cdot 2^1 < m! \cdot 2 \underbrace{<}_{\text{since } m \ge 4 \text{, so } 2 < m+1} m! \cdot (m+1) = (m+1)!$$

  • So by induction, $2^{n} < n!$ for all integers $n \ge 4$

Strong Induction Law

Let $S$ be a subset of $\mathbb{N}$, and suppose $S$ has the following true properties:

  1. $1 \in S$
  2. $\left( \{1, 2, 3, 4 \ldots n \} \subseteq S \right) \implies n + 1\in S$
    Then $S = \mathbb{N}$

Proof

Let $T = \{ k \in \mathbb{N}: \{1, 2, 3, 4 \ldots k\} \subseteq S\}$
(If $k \in T$, then $k \in S$, i.e. $T \subseteq S$)
$1 \in S$, so $\{1\} \subseteq S$, and so $1 \in T$
Now if $k \in T$, then $\{1, 2, 3 \ldots k \} \subseteq S$
So by 2. $k + 1 \in S$
But then $\{1, 2, 3 \ldots k , (k+1)\} \in S$, so $k + 1 \in T$
So $1 \in T$ and $(k \in T) \implies (k+1 \in T)$. So by Weak Induction, $T= \mathbb{N}$. It follows that $S=\mathbb{N}$

The difference between Weak Induction and Strong Induction is that Weak Induction only assumes $P(n)$ to be true and then shows that $P(n+1)$. However, Strong Induction has a “stronger” assumption by assuming that $P$ is true for all elements before $n$. Strong Induction assumes that $P(1), P(2), …P(n)$ are all true. Then Strong Induction shows that $P(n+1)$ is true based on this assumption.

Proposition

Let $n \in \mathbb{N}, \ n \neq 1$. Then $n$ is a product of prime numbers.

Proof

  • Base Case: $n = 2$
    $2$ is prime, so $2$ is its own prime factorization.

  • Inductive Hypothesis: Suppose the numbers $2, 3, 4, 5, 6\ldots m $ are all products of primes. We must show that $m + 1$ is also a product of primes.

  • Since $m + 1 > 1$ either $m+1$ is prime or $m+1$ is composite.

  • If $m+1$ is prime, it is its own prime factorization.

  • Else $m+1$ is composite.
    $$ \exists a, b \in \mathbb{N} \ \ s.t. m + 1 = ab $$
    $$ a \in \{2, 3, 4, \ldots m\}$$
    $$ b \in \{2, 3, 4, \ldots m\}$$

  • By IH (Inductive Hypothesis), $ a = \prod_{i=1}^{s} p_i$ and $b = \prod_{j=1}^{t} q_j$ where each $p_i$ and $q_j$ is prime.
    $$ m+1 = ab = \left( \prod_{i=1}^{s} p_i \right) \left( \prod_{j=1}^{t} q_j \right) $$

This is an example of where strong induction is useful and not weak.

4.1: The Language of Sets

  • A Set is a collection of objects
    • Order doesn’t matter
    • No duplicate elements
  • $x \in A$ means $X$ is an element of $A$
  • $\text{#}A$ is the cardinality (# of elements)

Set Builder Notation

$$ \{varabie: condition\} $$
$$ \{x \in A| P(x)\} $$

Subset

If $A$ and $B$ are sets, we write $A \subseteq B$ if $\forall x \in A, x \in B$

$$ A \subseteq B \land B \subseteq A \equiv A= B $$

$A \not\subseteq B$ means $\exists x \in A \ s.t. \ x \notin B$

If we want to prove $A \subseteq B$, then we can use the following skeleton:
Let $x\in A$

Then check that $x$ also satisfies the membership criteria of $B$
Hence $x \in B$

Example

  1. Let $A = \{x \in \mathbb{Z}: \exists a \in \mathbb{Z}$ so that $x = a^2\}$
    Let $B = \{x \in \mathbb{Z}: x \equiv 0 \pmod 4 \lor x \equiv 1 \pmod 4 \}$
    Prove $A \subseteq B$
    Let $x \in A$. Then $x\in \mathbb{Z}$, and $ \exists a \in \mathbb{Z}$ so that $ x = a^2$.
    By the division algorithm, we can write $a = 4q + r, \ \ q \in \mathbb{Z}, \ \ r \in \{0, 1, 2, 3\}$
    If $r = 0$, then $x = a^2 = \left( 4q \right)^2 = 4(4q^2) + 0$, so $x$ leaves remainder $0$ on division by $4$
    If $r = 1$, then $x = a^2 = \left( 4q + 1\right)^2 = 4(4q^2 + 2q) + 1$, so $x$ leaves remainder $1$ on division by $4$
    If $r = 2$, then $x = a^2 = \left( 4q + 2\right)^2 = 4(4q^2 + 4q + 1) $, so $x$ leaves remainder $0$ on division by $4$
    If $r = 3$, then $x = a^2 = \left( 4q + 3\right)^2 = 4(4q^2 + 6q + 2) + 1 $, so $x$ leaves remainder $1$ on division by $4$

    In each case $x \equiv_0 \pmod 4 \lor x \equiv 1 \pmod 4$. Thus $x \in B$, and it follows that $A \subseteq B$

  2. Let $A = \{x \in \mathbb{Z}: \exists a \in \mathbb{Z}$ so that $x = a^2\}$
    Let $B = \{x \in \mathbb{Z}: x \equiv 0 \pmod 4 \lor x \equiv 1 \pmod 4 \}$
    Prove $B \not\subseteq A$

    Consider $32 \in \mathbb{Z}$
    $ 32 = 4(8) + 0 $, so $32 \equiv 0 \pmod 4$, and we see that $32 \in B$
    $5^2 = 25 < 32$, and $6^2 = 36 > 32$, so $5 < \sqrt{32} < 6$, and $32 \not\subseteq A$

    1. Let $A$ and $B$ be sets. $A = B$, if $(A \subseteq B) \land \{B \subseteq A\}$
      Equivalent to $x \in A \iff x \in B $
      Let $C = \{x \in \mathbb{Z}: \text{x is odd}\} $
      Let $D = \{x \in \mathbb{Z}: x \equiv 1 \pmod 4 \lor x \equiv 3 \pmod 4\} $
      Prove $C = D$

    x \in \mathbb{Z}, $ and $\exists x = 2q + 1$. Prove that $x = 4t + 1 \lor x = 4+3$. Suppose by BWC, suppose that $x$ does not leave remainder 3 or 1. In the end, $x$ is found to be even, which is a contradiction since $x$ is supposed to be odd.

    Prove the converse:

  • Let $x \in D$. Then $\forall \mathbb{Z}$ and $ x \equiv 1 \pmod 4 \lor x \equiv 3 \pmod 4$
  • If $x \equiv 1 \pmod 4$, then $x = 4t+1$, for some $t \in 2$
  • $x$ is still odd
  • If $x \equiv 3\pmod 4$ and $x =4t + 3$, for some $t \in \mathbb{Z}$, $x$ is still odd. In both cases, $x \in C$

Proposition

Let $A, B, C$ be sets.
Suppose $A \subseteq B \land B \subseteq C$. Is it true that $A \subseteq C$?

What Sets Can’t Be

Let $S$ be a set
Call $S$ normal if $S \not\in S$
Let $X = \text{set of all normal sets}$
Is $x$ itself normal?

Midterm 1 Review

Problem 2

Prove that $6 | 13^n - 7^n$

Inductive Hypothesis

Strong Induction

Assume that for $\text{ } k = 1, 2, 3 \ldots m, \ \ 6 | 13^k - 7^k$,

Weak Induction

Assume that for some $ m \in \mathbb{N}, \ \ 6 | 13^m - 7^m$

Show that $6 | 13^{m+1}- 7^{m+1}$

Problem 3

Show by induction that $\forall n \in \mathbb{N} \ \ \sum_{i=1}^{n} i^2 = \frac{n(n+1)\left( 2n+1 \right) }{6}$

Base Case

$$ \sum_{i=1}^{1} i^2 = 1^2 = 1, \ \ \frac{1(1+1)(2(1) + 1}{6 }= 1$$

Inductive Hypothesis

Weak Induction

Suppose that for some $m \in \mathbb{N}, \ \ \sum_{i=1}^{m} i^2 = \frac{m(m+1)(2m+1)}{6}$
We will prove that $\sum_{i=1}^{m+1} i^2 = \frac{(m+1)(m+2)(2m+3}{6}$

4.2: Set Operations

Let $A$ and $B$ be sets.

Union

$$A \cup B = \{x: x \in A \lor x \in B\} $$

Commutative since or is commutative

Intersection

$$ A \cap B = \{x: x \in A \land x \in B\}$$

Commutative since and is commutative

Relative Complement or Set Difference

$$ A - B = \{x: x \in A \land x \neq B\}$$

Not commutative

Example

Let $A = \{0, \pi e, i, 1\}$
Let $B = \{\pi, e, \sqrt{2}, \sqrt{3} \}$

a. $A \cup B = \{0, \pi, e, i, 1, \sqrt{2} , \sqrt{3} \}$
b. $ A \cap B = \{\pi, e\} $
c. $ A - B = \{0, i, 1\}$

Universal Set

$U - A = A^{C}$

Proposition

  1. $A \cup B = B \cup A$
  2. $a \cap B = B \cap A$
  3. $(A \cup B)\cup C = A \cup (B\cup C)$
  4. $(A \cap B) \cap C = A \cap (B\cap C)$
  5. $A \cap \emptyset = \emptyset$
  6. $A \cup \emptyset = A$
  7. $A \cup (B\cap C) = (A \cup B) \cap (A \cup C)$
  8. $A \cap (B \cup C) = (A \cap B) \cup (A \cap C)$

Proof of 8

$$ A \cap (B \cup C) = \{x: x \in A \land x \in B \cup C\}$$
$$ = \{x: x \in A \land (x \in B \lor x \in C)\}$$
$$ = \{x: (x \in A \land x \in B) \lor (x \in A \land x \in C)\}$$
$$ = (A \cap B) \cup (A \cap C)$$

Cardinality

If $A$ and $B$ are finite sets, then $\text{#}(A \cup B) = \text{#} A + \text{#} B - \text{#} (A \cup B)$

Cartesian Product

The Cartesian product $A \cross B$:
$$ A \cross B = \{(a, b): a \in A, b \in B\}$$

Power Set

If $X$ is a set, then $P(X) = \{A: A \subseteq X\}$ is called the power set

Example

$$x = \{a, b\}$$
$$P(X) = \{\emptyset, \{a\}, \{b\}, \{a, b\}\}$$

Proposition

Let $A$ and $B$ be sets.
$$ (A \subseteq B) \iff \left( P(A) \subseteq P(B) \right) $$

Proof:

Prove $(A \subseteq B) \implies \left( P(A) \subseteq P(B) \right)$
$$ C\in P(A) \implies C \subseteq A $$
$$ A \subseteq B\implies C \subseteq B \text{ by transitivity}$$
$$ C \subseteq B \implies C \in P(B)$$

Prove the converse
$$ A \subseteq A \implies A \in P(A)$$
$$ A \in P(B) \implies A \subseteq B $$

DeMorgan’s Laws

  1. $ A - (B \cup C) = (A - B) \cap (A - C) $
  2. $A - (B \cap C) = (A - B) \cup (A - C)$

Proof of 1

$$ x \in A \land \left( x \notin \left( B \cup C \right) \right) $$
$$ x \in A \land \neg\left( x \in B \lor x \in C \right) $$
$$ x \in A \land x \notin B \land x \notin C$$
$$ \left( x \in A \land x \notin B \right) \land \left( x\not\in C \land x \in A \right) $$
$$ A - B \cap A - C$$

Converse
$$ \left( A - B \right) \cap \left( A - C \right) $$
$$ x \in \left( A - B \right) \land x \in \left( A - C \right) $$
$$ x \in A \land x \not \in B \land x \in A \land x \notin C$$
$$x\in A \land \neg\left( x \in b\lor x \in C \right) $$
$$ x \in A \land x \not\in \left( B \cup C \right) $$
$$ A - B\cup C$$

4.3: Arbitrary Unions and Intersections

$$ \left( A \cup B \right) \cup C = A \cup \left( B \cup C \right) $$
$$ \left( A \cap B \right) \cap C = A \cap \left( B \cap C \right) $$
so we can just write $A \cap B \cap C$ and $A \cup B \cup C$ without parentheses.

Definitions

Let $A_1, A_2, A_3, \ldots, A_n, \ \ \ n \in \mathbb{N}$ be sets

  1. $\bigcap_{i=1}^{n} A_i = A_1 \cap A_2 \cap A_3 \cap \ldots \cap A_n $
  2. $\bigcup_{i=1}^{n} A_i = A_1 \cup A_2 \cup A_3 \cup \ldots \cup A_n $

Proposition

Let $n \in \mathbb{N}$, and $B_1, B_2, B_3, \ldots B_n$ be sets.

  1. $A \cap \left( \bigcap_{i=1}^{n} B_i \right) = \bigcap_{i=1}^{n} \left( A \cap B_i \right) $
  2. $A \cup \left( \bigcup_{i=1}^{n} B_i \right) = \bigcup_{i=1}^{n} \left( A \cup B_i \right) $
  3. $A - \left( \bigcup_{i=1}^{n} B_i \right) = \bigcap_{i=1}^{n} \left( A - B_i \right) $
  4. $A - \left( \bigcap_{i=1}^{n} B_i \right) = \bigcup_{i=1}^{n} \left( A - B_i \right) $

Proof of 2

Base Case: $n = 1$
$$ \bigcap_{i=1}^{1} \left( A \cup B_i \right) = A \cup B_1 = A \cup \left( \bigcap_{i=1}^{1} B_i \right) $$

Inductive Hypothesis:
Suppose that for some $m \in \mathbb{N}$, $A \cup \left( \bigcap_{i=1}^{m} B_i \right) $ holds
Let $B_1, B_2, B_3, \ldots B_n$ be sets
$$ A \cup \left( \bigcap_{i=1}^{m+1} B_i \right) = A \cup \left( \bigcap_{i=1}^{m} B_i \cap B_{m+1} \right) $$
$$ A \cup \left( \bigcap_{i=1}^{m} B_i \right) \cap \left( A \cup B_{m+1} \right) $$
$$ \bigcap_{i=1}^{m} \left( A \cup B_i \right) \cap \left( A \cup B_{m+1} \right) = \bigcap_{i=1}^{m+1} \left( A \cup B_i \right) $$

5.3: Injective and Surjective Functions

Definition

Let $X, Y$ be sets, and let $f: X \to Y$

  1. $f$ is one-to-one (1-1) or injective if
    $$
    \left( \forall x_1, x_2 \in X \right) [f(x_1) = f(x_2) \implies x_1 = x22]
    $$
    the contrapositive of the above is also true

  2. $f$ is onto or surjective if
    $$
    (\forall y \in Y) \left( \exists x \in X \right) [y = f(x)]
    $$

  3. $f$ is bijective $f$ is both injective and bijective.

Theorem 5.3.10

Let $X, Y, Z$ be sets. Let $f: X \to Y$ and $g: Y \to Z $

  1. If $f$ and $g$ are both 1-1, then $g \circ f$ is also 1-1
  2. If $f$ and $g$ are both onto, then $g \circ f$ is also onto
  3. If $f$ and $g$ are both bijections, then $g \circ f$ is also a bijection
  4. If $g \circ f$ is 1-1, then $f$ is 1-1, but $g$ is not necessarily 1-1
  5. If $g \circ f$ is onto, then $g$ is onto, but $f$ is not necessarily onto

5.4: Invertible Functions

Definition

Let $X, Y$ be sets, and let $f: X \to Y$. We say that $f$ is invertible if there exists a function $g: Y \to X$ such that for all $x\in X$ and for all $y \in Y$
$$
y = f(x) \iff x = g(y)
$$

Proposition 5.4.3

Let $X$ and $Y$ be sets, and let $f: X \to Y$ and $g: Y \to X$. Then $f$ is invertible and $g$ is an inverse function of $f$ iff $g \circ f = I_X$ and $f \circ g = I_Y$

Theorem 5.4.7

Let $X, Y$ be sets and let $f: X \to Y$

  1. $f$ is invertible iff $f$ is a bijection
  2. If $f$ is invertible, then its inverse function is unique

7.1: Relations

Definitions

Let $A$ and $B$ be sets, and let $R \subseteq A \cross B$ be a relation from $A$ to $B$. For $a \in A$ and $b \in B$,

$$ a R b \ \ \ \text{ iff } (a, b) \in R $$
$$ a \not R b \ \ \ \text{ iff } (a, b) \not\in R $$

7.2: Equivalence Relations

Definition

Let $\sim$ is a relation on a set $A$

  1. $\sim$ is reflexive if $\left( \forall a \in A \right)\left( a \sim a \right) $
  2. $\sim$ is symmetric if $\left( \forall a, b \in A \right) \left( a \sim b \implies b \sim a \right) $
  3. $\sim$ is transitive if $\left( \forall a, b, c \in A \right) [\left( a \sim b \land b \sim c \right) \implies a \sim c] $
  4. $\sim$ is an equivalence relation if $\sim$ is reflexive, symmetric, and transitive

8.1: Introduction to Finite and Infinite Sets

$$A_m = \{1, 2, …, m\} = \{i \in \mathbb{N} : i \le m\} $$
$$ A_0 = \emptyset $$

Definition 8.1.2

  1. $X$ is equinumerous with $Y$, denoted $X \approx Y$, if there exists a bijection $f: X \to Y$

  2. $X$ is finite if $X = \emptyset$ or there exists $m\in \mathbb{N}$ such that $A_m \approx X$. In other words there exists a bijection $f: \{1, 2, 3, \ldots, m\} \to X$

  3. If $A_m \approx X$, then the cardinality of $X$ is $n$, denoted as $\#X= n$. We define $\#\emptyset = 0$

  4. We say $X$ is infinite if $X$ is not finite.

$\approx$ is an equivalence relation on the collection of all nonempty sets.

Example

Show $\mathbb{N} \approx \mathbb{Z}$

First, define $f: \mathbb{N}\to \mathbb{Z}$ for all $n \in \mathbb{N}$
$$
f(n) = \begin{cases}
\frac{n}{2} & \text{ n is even } \\
\frac{-\left( n-1 \right) }{2} & \text{ n is odd }
\end{cases}
$$

Show $f$ is 1-1.
Define $ n_1, n_2 \in \mathbb{N}$ and suppose $f(n_2) = f(n_1)$
If both are even, then
$$\frac{n_1}{2}= \frac{n_2}{2} \implies n_1 =n_2 $$
If both $ n_1$ and $ n_2$ are odd,
$$\frac{-\left( n_1-1 \right) }{2} = \frac{-\left( n_2-1 \right) }{2} $$
One can’t be odd and one even.

Show $f$ is onto.
Suppose $n \in \mathbb{Z}$ If $n \ge 1$, then $$
f(2n) = \frac{2n}{2} = n
$$
If $n\le 0$
$$
f(-2n+1) = \frac{-\left( -2n+1 \right) -1}{2} = n
$$
$ n \in im(f) \ \forall n \in \mathbb{Z}$, so $f$ is onto.

8.2: Finite Sets

Recall: Let $A$ be a set. $\forall n \in \mathbb{N}$, let $I_n = \{1, 2, 3, …n\}$

  1. We say $A$ is finite if $A = \emptyset$ or $\exists n \in N \ s.t. \ \exists f: A \rightarrow I_n$ (bijection)
    $$\# A = 0 \text{ or } \#A = n$$

  2. Every subset of a finite set is finite.

  3. If $A$ and $B$ are finite sets, and $A \cup B = \emptyset$, then $\#(A \cup B) = \#A + \#B$

Pigeonhole Principle

If $n +1 $ or more pigeons are placed into $n$ pigeonholes, then at least one pigeonhole will contain more than 1 pigeon

Example 1

Choose 5 points interior to a 2m by 2m square. Prove that among these 5 there is a pair such that the distance between the two points in the pair is $\le \sqrt{2}$ m.

Answer: Divide square into four $1 \cross 1$ square. Notice that there must be one square with 2 points. The greatest distance between any two points in one of the sub squares is just the diagonal $\sqrt{2}$.

Example 2

Show there is a multiple of $7$ whose decimal digits are all 1’s or 0’s

Consider $1, 11, 111, 1111, 11111, 111111, 1111111, 11111111$
2 numbers on this list must leave the same remainder upon division by $7$

Say there are

$$ a_j = \underbrace{111…1}_{\text{j-ones}} $$

$$ a_k = \underbrace{111…1}_{\text{k-ones}} \ \ \ \ k > j$$

Then $7 | a_k - a_j$ since $a_k \equiv a_j \mod 7$
Thus the $a_k - a_j$ have only 0 and 1 as digits.

More on Finite Sets

  1. Suppose $A_1, A_2, A_3, …, A_n$ are pairwise disjoint sets ($A_i \cap A_j = \emptyset, \text{ whenever } i \neq j$)
    Then $\#\left(\cup_{i=1}^{n} A_i\right) = \sum_{i=1}^{n} \left(\#A_i\right)$

  2. Let $A$ and $B$ be finite sets. Then $\# (A \cup B) = \#A + \#B - \#(A \cap B)$

$$ \#(A \cross B) = (\#A) \cdot (\#B)$$

8.3: Infinite Sets

Definitions

  1. The set $X$ is denumerable or enumerable or countably infinite if there is a bijection $f: \mathbb{N} \to X$ and $f$ is onto and 1-1.
    $$ \mathbb{N} \approx X$$
  2. The set $X$ is countable if $X$ is finite or denumerable.
  3. The set $X$ is uncountable if $X$ is not countable, if $X$ is infinite and not denumerable

Theorem

Let $A$ be a nonempty set. Suppose $g: \mathbb{N} \to A$ is surjective. Then $A$ is countable.

Proof

Either $A$ is finite or it is not

If $A$ is finite, there is nothing to prove, since $A$ is obviously countable. So suppose $A$ is infinite.

We must construct a bijection from $\mathbb{N} \to A$ to show that $A$ is countable (countably infinite).

Put $f(1) = g(1)$

Suppose inductively, that we have defined $f(1), f(2), f(3), …, f(n)$ so that $f(i) \neq f(j)$ whenever $i \neq j$

$$i, j \in \{1, 2, 3, …, n\}$$
$$\{ g(1), g(2), g(3), …, g(n) \} \subseteq \{f(1), f(2), f(3), …, f(n) \}$$

Define $f(n+1)$

$B_n = \{f(1), f(2), f(3)…, f(n)\}$ is finite and $A$ is infinite, so $a \ B$ is nonempty

Then $X = \{m\in \mathbb{N}: g(m) \notin B_n \}$ is also nonempty, since $g$ is surjective.

Then $X$ is a nonempty subset of $\mathbb{N}$. Then by the Well-Ordering Principle, $X$ has a least element, say $m_0 = min(X)$

$$\text{Let } f(n+1) = g(m_0)$$
$$ f(n+1) \neq f(i) \ \forall i \in \{1, 2, 3, .., n\}$$

$$ \{g(1), …, g(n)\} \subseteq B_n \implies m_0 \ge n+1$$

$$ g(n+1) \in \{f(1), f(2), f(3), …, f(n), f(n+1)\}$$

$$\forall a, b \in \mathbb{N} \ , a < b \implies f(a) \neq f(b) $$
So $f$ is injective.

We have defined an injective function $f$ so that we don’t have “duplicate” values of $g(x)$ ($A$) in the codomain of $f$

Let $c \in A$
$$g \text{ is surjective } \implies \exists x \in \mathbb{N} \text{ s.t. } c= g(x)$$
$$ \{g(1), g(2), …, g(x)\} \subseteq \{f(1), f(2), f(3), …, f(x)\} \implies \exists t \in \{1, 2, 3, …, x\} \text{ s.t. } g(x) = f(t)$$

$$ c\in img(f) \implies f \text{ is surjective }$$

Proposition

Let $A$ be a nonempty countable set. Then $\exists g : \mathbb{N} \to A$ such that $g$ is surjective.

Proof

Note: $I_m = \{1, 2, 3, …, m\}$

If $A$ is countably infinite, then $\exists f: A \to \mathbb{N}$ which is bijective. Then $f^{-1}: \mathbb{N} \to A$ is also bijective and thus surjective.

If $A$ is finite, then $\exists f: I_m \to A$ for some $m \in \mathbb{N}$
$$ g: \mathbb{N} \to A$$
$$ x \to \begin{cases} f(x) & 1 \le x \le m \\ f(1) & x > m \end{cases}$$

Proposition

Let $A$ and $B$ be denumerable sets (countably infinite). Then $A \cup B$ is also enumerable.

Proof

If $A$ and $B$ are denumerable, then $\exists f: \mathbb{N} \to A$ and $\exists g: \mathbb{N} \to B$ which are also bijections.

Define the following:
$$ h: \mathbb{N} \to A \cup B $$
$$ x \to \begin{cases} f(x/2) & \text{ x is even} \\ g(\frac{x+1}{2}) & \text{x is odd}\end{cases}$$

Let $y \in A\cup B$

If $y \in A$, then $y=f(x)$ for some $x \in \mathbb{N}$, ($f$ is surjective)
$$ h(2x) = f\left(\frac{2x}{2}\right) = f(x) = y$$

If $y \in B$, then $y=g(x)$ for some $x \in \mathbb{N}$, ($g$ is surjective)
$$ h(2x-1) = f\left(\frac{(2x-1)+1}{2}\right) = g(x) = y$$

In either case, $g(x) = y$, so $y\in im(h)$, and $h$ is surjective.

By the previous theorem, $A \cup B$ is denumerable.

Corollary

If $A_1, A_2, A_3, .., A_n$ are all countable, then $\cup_{i=1}^{n} A_i$ is also countable.

Proposition

$\mathbb{N} \cross \mathbb{N}$ is countable

$$ g: \mathbb{N} \to \mathbb{N} \cross \mathbb{N}$$
$$ x \to \begin{cases} (m,n) & \text{ if } x = 2^m 3^n \\ (1, 1) & \text{ else} \end{cases}$$

Fundamental Theorem of Arithmetic

Let $n \in \mathbb{N} - \{1\}$. Then $n$ is a “product” of prime numbers (i.e. $n$ is either prime itself or a product of primes). Moreover, this prime factorization is unique.

Proof (Existence)

We can use induction to prove the existence of a prime factorization.

See this proof

Proof (Uniqueness)

Lemma: Let $a_1, a_2, a_3, …, a_n \in \mathbb{Z}$. Let $p$ be prime and suppose $p | \left(\prod\limits_{i=1}^{n} a_i\right)$, then $p|a_i$ for some $i \in \{1, 2, 3, …, n\}$. (Based on Euclid’s Lemma by induction)

Now suppose $n = p_1 \cdot p_2 \cdot p_3… \cdot p_s$ and $n = q_1 \cdot q_2 \cdot q_3… \cdot q_s$ are both prime factorizations of $n$ ($q_i$ and $p_i$ are always prime).

Assume without loss of generality (WLOG), $s \le t$

$$ p_1(p_2\ p_3 ...p_s) = q_1 \ q_2 \ q_3 … q_t$$
$$ p_1 | q_1\ q_2\ q_3 … \ q_t$$

By Euclid’s Lemma $p_1 | q_i$ for some $i \in \{1,2, 3, …, t\}$. By reordering the $q_i$'s, WLOG, we can assume that $p_1 | q_1$.
But $p_1 = q_1$, since the only positive integer factors of $q_1$ are $1$ and $q_1$, but $p_1$ is prime $\implies p_1 \neq 1$.

$$ p_1\ p_2\ p_3 ...p_s = q_1 \ q_2 \ q_3 … q_t$$
$p_1 = q_1 $, so
$$ p_2\ p_3 ...p_s = q_2 \ q_3 … q_t$$

We can use the same technique to show that $p_2 = q_2$, $p_3 = q_3$, and so on.

After $s$ repetitions, we get $p_i = q_i$ for $i = 1, 2, 3, …, s$

Now we need to show that $s = t$

We assumed $s\le t$. Suppose BWOC $s<t$

$$ p_1 \ p_2 … \ p_s = n = q_1 \ q_2 \ q_3 … \ q_s \ q_{s+1} \ q_{s+2} … \ q_t $$
But $p_i = q_i$ for $i \in \{1, 2, …, s\}$, so
$$ 1 = q_{s+1}\ q_{s+2} \ … q_t$$
This is a impossible since $q_j \ge 2$ for all $j$. So $s = t$

Proof that $\ \mathbb{Q} \ $ is countable

Proposition

$\mathbb{N} \cross \mathbb{N}$ is countable

Proof

Define $g: \mathbb{N} \to \mathbb{N} \cross \mathbb{N}$

$$ x \to \begin{cases} (m, n) & x = 2^m3^n \\ (1, 1) & else \end{cases}$$

Example: $$g(96) = g(2^5 3^1) = (5, 1)$$

Uniqueness of prime factorization guarantees there’s only one way to write $x$ as a prime factorization of $2$ and $3$, so this means the function is well defined. Let $(x, y) \in \mathbb{N} \cross \mathbb{N}$. Then $g(2^x 3^y) = (x, y)$, so $g$ is surjective.

Continuing Proof

Now let $\mathbb{Q}^+ = \{\frac{a}{b}: a, b \in \mathbb{N}\}$
$$ h: \mathbb{N} \cross \mathbb{N} \to \mathbb{Q}^+$$
$$(a, b) \to \frac{a}{b}$$
$h$ is surjective. Then $(h \circ g): \mathbb{N} \to \mathbb{Q}$ is surjective (composition of surjective functions is surjective). So $\mathbb{Q}^+$ is countable.

Put $\mathbb{Q}^- = \{-\frac{a}{b}: a, b \in \mathbb{N}\}$
$\mathbb{Q}^-\approx \mathbb{Q}^+$, so $\mathbb{Q}^-$ is also countable. (Construct bijection between them to show that they are equiponent (e.g. $f(x) = -x$))

$\{0\}$ is clearly countable.

$$ \mathbb{Q} = \mathbb{Q}^- \cup \{0\} \cup \mathbb{Q}^+$$
So $\mathbb{Q}$ is countable!

Example

Is it true that if $A$ and $B$ are countable sets, then so is $A \cross B$?

If $A = \emptyset \lor B = \emptyset$ , then answer is yes, since $A \cross B = \emptyset$. So assume $A$ and $B$ are nonempty.

Since $A$ and $B$ are also countable, there are surjections $f: \mathbb{N} \to A$ and $g: \mathbb{N} \to B$

We will use the fact that $\mathbb{N} \cross \mathbb{N}$ is countable.

$$ h: \mathbb{N}\cross \mathbb{N}\to A \cross B$$
$$ (x, y) \to (f(x), g(y))$$

h is surjective:
$$ \text{Let } (a, b) \in A \cross B$$
$$ \text{f is surjective } \implies( \exists c \in \mathbb{N}) (f(c) = a)$$
$$ \text{g is surjective } \implies( \exists d \in \mathbb{N}) (f(d) = b)$$

$$h((c, d)) = (f(c), g(d)) = (a, b) $$
$$ (a, b) \in im(h)$$
So $h$ is surjective.

Now since $\mathbb{N}\cross\mathbb{N}$ is countable, there is a surjection
$$t: \mathbb{N} \to \mathbb{N}\cross\mathbb{N}$$

$$ h\circ t: \mathbb{N} \to A \cross B$$
$h\circ t$ is also a surjection, so $A \cross B$ is also countable.

Proposition

Let $A$ and $B$ be sets with $A$ being countable. Let $B \subseteq A$. Then $B$ is also countable.

Proof

If $B = \emptyset$, then $B$ is finite and thus countable. Assume $B \neq \emptyset$

Since $A$ is countable, there is a surjection
$$ f: \mathbb{N}\to A$$

Fix $b \in B$. Define
$$
g: \mathbb{N}\to B
$$
$$
n \to \begin{cases}
f(n) & f(n) \in B \\
b & f(n) \not\in B
\end{cases}
$$
Let $b \in B$. Since $f$ is surjective, $\exists x \in \mathbb{N}$ with
$$ f(x) = y\in B $$

Then $g(x) = f(x) = y$, so $y \in im(g)$, so $g$ is surjective.

$$ \text{B is countable}$$

Corollary

Let $X$ be a set and suppose $Y \subseteq X$

If $Y$ is uncountable, then $X$ is uncountable.

Theorem (Cantor): $[0, 1) \ $ is uncountable.

Proof

Suppose BWOC, that $[0, 1)$ is countable.

In this case, there would exist a surjection $f: \mathbb{N}\to [0, 1)$
Super scripts are not exponents, 0.abcd denotes decimal expansion of a numer
$$
f(2) = 0.d_1^1 d_2^1 d_3^1 \ldots
$$
$$
f(2) = 0.d_1^2 d_2^2 d_3^2 \ldots
$$
$$
f(3) = 0.d_1^3 d_2^3 d_3^3 \ldots
$$
$$
f(4) = 0.d_1^4 d_2^4 d_3^4 \ldots
$$
$$
\\
d_i^t \in \{0, 1, 2, 3, \ldots 9\}
$$

Consider the following number
$$
r =0.r_1r_2r_3r_4 \ldots
$$
$$
r = \begin{cases}
4 & d_i^i \neq 4 \\
5 & d_i^i \neq 4
\end{cases}
$$

$$
(\forall n \in \mathbb{N})\left( f(n) \neq r_n \right)
$$
since
$$
\left( \forall n\in\mathbb{N} \right) r_n \neq d_n^n
$$

So $r \not\in im(f)$, contradicting surjectivity of $f$.

Hence $[0, 1)$ is uncountable.

Corollary

$\mathbb{R}$ is uncountable

Proof

$[0, 1) \subseteq \mathbb{R}$

Groups

Definitions

Let $G$ be a set.

An operation, $\star$, on $G$ is a function $\star: G \cross G \to X$
where $X$ is a set. We write $g \star h$ instead of $\star(g, h)$.

We say that $\star$ is closed if $im(\star) \subseteq G$

Addition is closed
$$ +: \mathbb{Z} \cross \mathbb{Z} $$
Multiplication is closed
$$ \cdot : \mathbb{Z} \cross \mathbb{Z} $$
Division is not closed
$$ /: (\mathbb{Z} - \{0\}) \cross \left( \mathbb{Z} - \{0\} \right) $$

Identity

$$ a \star x = b$$
$$ a^{-1} \star (a \star x) = a^{-1} \star b$$
$$ (a^{-1} \star a) \star x = a^{-1} \star b$$
$$
e \star x = a^{-1} \star b
$$
where $e$ is the identity element
$$
x = a^{-1} \star b
$$

Property of a Group

Let $G$ be a set and let $\star$ be a closed operation on $G$.

We say that $(G, \star)$ is a group if

  1. Commutative: $(\forall a, b, c \in G) \left( a \star \left( b \star c \right) = \left( a \star b \right) \star c \right) $
  2. Identity Element: $(\exists e \in G) \left( \forall a \in G \ \ \ a \star e = e \star a = a \right) $
  3. Inverse: $(\forall a \in G \ \ \ \exists a^{-1} \in G )\left( a \star a^{-1} = a^{-1} \star a = e \right) $
  4. (If $\forall a, b, \in G \ \ \ a \star b = b \star a$, then the group is an abellian group)

Examples of a Group

  1. $(\mathbb{Z}, +)$
  2. $\left( \mathbb{Q} - \{0\}, \cdot \right) $
  3. $\left( V, +\right) $, where $V$ is only $\mathbb{R}$-vector space
  4. $\left( \mathbb{R}, + \right) $
  5. $\left( \mathbb{R} - \{0\}, \cdot \right) $

Is the Identity Element Unique?

Suppose that
$$\forall a \in G \ \ \ a \star e = e \star a = a$$
$$
\forall a \in G \ \ \ a \star d = d \star a = a
$$
$$
d \star e = d
$$

$$
d \star e = e
$$

$$
d = e
$$
Thus the identity is unique.

Is the Inverse Unique

Suppose $b \star a = a \star b = e$ and $c \star a = a \star c = e$ (i.e. both $b$ and $c$ are inverses of $a$

$$ b \star a \star c = \left( b \star a \right) \star c = e \star c = c$$
$$ b \star a \star c = b \star \left( a \star c \right) = b \star e = b$$

$$ \implies b = c $$
$a^{-1}$ is unique

Notation

We often just write “let $G$ be a group” and use multiplication for the operation on $G$, even if the operation isn’t traditional multiplication

$ab$ instead of $a \star b$
$a + b$ instead of $a \star b$ sometimes when group is abelian

Examples of Finite Groups

Fix $n \in \mathbb{N}$. Then $\mathbb{Z} / n \mathbb{Z} = \{[0], [1], [2], \ldots [n-1] \}$ with
$$ [a] + [b] = [a + b]$$
forms a group

  • $[0]$ is the identity
  • $[-a]$ is the inverse of $a$

Example of Non-Abelian Groups

$$ G = \left\{\begin{bmatrix} a & b \\ c & d \end{bmatrix}: ab - bc \neq 0 \right\}$$

Example of Non-Abelian Groups

Let $X = \{1, 2, 3\}$
Let $S_3 = \{f: f: X \to X \text{ and f is bijective}\}$

$$ f_1 = \begin{pmatrix} 1 & 2 & 3 \\ 1 & 2 & 3 \end{pmatrix} $$
means $f_1(1) = 1, \ \ f_1(2) = 2, \ \ f_1(3) = 3$

$$ f_2 = \begin{pmatrix} 1 & 2 & 3 \\ 1 & 3 & 2 \end{pmatrix} $$
means $f_2(1) = 1, \ \ f_1(2) = 3, \ \ f_1(3) = 2$

$$ f_3 = \begin{pmatrix} 1 & 2 & 3 \\ 2 & 3 & 1 \end{pmatrix} $$
$$ f_4 = \begin{pmatrix} 1 & 2 & 3 \\ 3 & 2 & 1 \end{pmatrix} $$
$$ f_5 = \begin{pmatrix} 1 & 2 & 3 \\ 3 & 1 & 2 \end{pmatrix} $$
$$ f_6 = \begin{pmatrix} 1 & 2 & 3 \\ 2 & 1 & 3 \end{pmatrix} $$

$$
f_1, f_2, f_3, f_4, f_5, f_6 \in S_3
$$
If $f, g \in S_3$
$f \circ g = g \circ f$ is not true in general, so the group is not abelian.

More generally, if $X$ is any nonempty set,
$$ Sym(X) = \{f: f: X \to X, \ \ f \text{ is bijective }\}$$
is the symmetric group with composition of functions as its operation.

More Definitions

  • If $\left( G, \star \right) $ is a group and $H \subseteq G$ is nonempty, we say that $H$ is a subgroup of $G$ if the restriction of $\star$ to $H \cross H$ makes $H$ into a group itself.
  • Equivalently, if $G$ is a group and $H \subseteq G$ is nonempty, then $H$ is a subgroup if $\forall a, b \in H, \ ab^{-1} \in H $

Homomorphisms and Isomorphisms

Let $G_1$ and $G_2$ be groups. We say that $\phi: G_1 \to G_2$ is a homomorphism if
$$\forall a, b, \in G_1 \ \ \ \ \phi(ab) = \phi(a) \cdot \phi(b)$$

If $\phi$ is also bijective, it is instead called an isomorphism